You are on page 1of 1

MA 101

Calculus
Tutorial–1
(1) Find infimun (greatest lower bound) and supremum (lowest upper bound) of the fol-
lowing sets, if it exists.
(a) S = {x ∈ R : x > 0}.
(b) S = {1/n : n ∈ N}.
(c) S = {1 − (−1)n /n : n ∈ N}.
(d) S = {1/n − 1/m : n, m ∈ N}.
m
(e) S = { : m, n ∈ N}.
m+n
1
(2) Let A be a non-empty subset of R and α ∈ R. Show that α = sup A iff α − is not an
n
1
upper bound of A but α + is an upper bound of A for every n ∈ N.
n
(3) Let S be a non-empty subset of R that is bounded below. Prove that inf S = − sup{−s :
s ∈ S}.
(4) Investigate the convergence of the sequence (xn ) where
1 2 n
(a) xn = + +···+ .
1+n 2 2+n 2 n + n2
(b) xn = 1 + (−1)n .
1
(c) xn = .
n(1 + n)
2n
(d) xn = .
1 +n 
π 1
(e) xn = sin + .
2 n
n
X 1
(f) xn = √ .
k=1
n +k
2

(5) Let x0 ∈ Q. Show that there exists a sequence (xn ) of irrational numbers such that
xn → x0 .
(6) Let A := {an : n ∈ N} and B := {bn : n ∈ N} such that an < bn , ∀n ∈ N . If lowest
upper bound of A be ζ and greatest lower bound of B be η . If the interval In = [an , bn ]

\
is a sequence of closed and bounded nested intervals then [ζ, η] = In .
n=1

You might also like